Akademisyenler öncülüğünde matematik/fizik/bilgisayar bilimleri soru cevap platformu
1 beğenilme 0 beğenilmeme
34.7k kez görüntülendi

$a>0$  ve

$\lim_{x \rightarrow \infty}\left [\sqrt{ax^2+bx+c}-\sqrt{kx^2+zx+s}\right]$ gibi

$\lim_{x \rightarrow \infty}\kappa(x)-\delta(x)$  varsa 

$\infty - \infty$ belirsizliğini ortadan kaldırmak için

$\lim_{x \rightarrow \infty}\sqrt{ax^2+bx+c}=\lim_{x \rightarrow \infty}\left [ \sqrt a.(x+\frac{b}{2a}) \right]$

formulü kullanılır.

bu formülü ispatlayınız.

Orta Öğretim Matematik kategorisinde (7.8k puan) tarafından 
tarafından düzenlendi | 34.7k kez görüntülendi

2 Cevaplar

1 beğenilme 0 beğenilmeme
En İyi Cevap

$a>0$ olsun.

$$\lim\limits_{x\to \infty}\sqrt{ax^2+bx+c}$$

$$=$$

$$\lim\limits_{x\to \infty}\sqrt{a\left(x^2+\frac{b}{a}x+\frac{b^2}{4a^2}+\frac{c}{a}-\frac{b^2}{4a^2}\right)}$$

$$=$$

$$\lim\limits_{x\to \infty}\sqrt{a\left[\left(x^2+\frac{b}{a}x+\frac{b^2}{4a^2}\right)+\left(\frac{4ac}{4a^2}-\frac{b^2}{4a^2}\right)\right]}$$

$$=$$

$$\lim\limits_{x\to \infty}\sqrt{a\left[\left(x+\frac{b}{2a}\right)^2+\left(\frac{4ac-b^2}{4a^2}\right)\right]}$$

$$=$$

$$\lim\limits_{x\to \infty}\sqrt{a}\left(x+\frac{b}{2a}\right)\sqrt{1+\frac{\left(\frac{4ac-b^2}{4a^2}\right)}{a\left(x+\frac{b}{2a}\right)^2}}$$

$$=$$

$$\lim\limits_{x\to \infty}\sqrt{a}\left(x+\frac{b}{2a}\right)\cdot\underset{1}{\underbrace{\lim\limits_{x\to \infty}\sqrt{1+\frac{\left(\frac{4ac-b^2}{4a^2}\right)}{a\left(x+\frac{b}{2a}\right)^2}}}}$$

$$=$$

$$\lim\limits_{x\to \infty}\sqrt{a}\left(x+\frac{b}{2a}\right).$$

(11.4k puan) tarafından 
tarafından düzenlendi
0 beğenilme 0 beğenilmeme

Bu soru yanlis ifade edilmis. Yani ifade edilmek istendigi gibi olmamis. 

Zaten sol taraf sonsuza gidiyor. Sag tarafa $x$ de yazabiliriz sadece. O da sonsuza gider.

Soru $f-g \to 0$ seklinde sorulmaliydi bence. Bu durumda $f=g+(f-g)$ olarak limitlerin icerisinde degisiklik yaptigimizda $f-g \to 0$  olacagindan limitli islemlerde bu donusum ile $f$'in yerine bi nevi $g$ yazmis oluruz.

(25.3k puan) tarafından 

aynen dedıgınız gıbı bu notu eklemem gerek .

"Regularization" mi ne vardi, ben pek bilmiyorum ama bu tarz yerlerde kullaniliyor galiba. Mesela 1+2+3+...=-1/12 olmasi. Bu gercekte yanlis ama toplam gercekten varsa gibi belirli sartlar altinda kullanabiliyoruz bu esitligi. 

Burda da su denilmeli en azindan. Benin dedigim sekilde ispatlanip limit oldugu yerlerde bunlar arasinda degisiklik yapmamizin sonucu degistirmeyecegi gosterilmeli. 

Yoksa $x$ yazip sag tarafa, boyle bir degisiklik de yapabiliriz. Bu da hata getirir.

20,200 soru
21,726 cevap
73,275 yorum
1,887,787 kullanıcı